0 Daumen
1,1k Aufrufe



ich habe zu folgender Aufgabenstellung ein paar Fragen.

Ich soll sowohl bei a), als auch bei b) beweisen, dass es sich um eine Basis handelt.

An dieser Stelle würde ich die Polynome einfach als Vektoren darstellen und diese auf lineare unabhängigkeit überprüfen. Nun noch zeigen, dass es sich um ein Erzeugendessytem handelt und der Teil der Aufgabe wäre doch erledigt, oder etwa nicht?

Habe ich irgendetwas besonderes zu beachten, wenn ich zeigen will, dass es sich um eine Basis eines Untervektorraums handelt, anstatt um eine basis des vektorraums?

Handelt es sich bei F2 um die Rechenvorschrift modulo 2?

Vielen Dank schon mal :)

(\textbf{schriftlich, 7 Punkte})  \begin{itemize} \item[a)] Sei $V\subset \mathbb{F}_2[t]$ der $\mathbb{F}_2$-Untervektorraum der Polynome mit Koeffizienten in $\mathbb{F}_2$ vom Grad höchstens $2$. Beweisen Sie, dass $\mathcal{B}=\{\bar{1}+t, t+t^2, t^2\}$ eine Basis von $V$ ist und schreiben Sie den Vektor $p(t)=\bar{1}+t^2$ als Linearkombination von Elementen aus $B$. \item[b)] Sei $W\subset \mathbb{R}[t]$ der $\mathbb{R}$-Untervektorraum der Polynome mit Koeffizienten in $\mathbb{R}$ vom Grad höchstens $2$. Beweisen Sie, dass $\mathcal{A}=\{1+t+t^2, 2t-t^2, 2+t+t^2\}$ eine Basis von $W$ ist und schreiben Sie die Vektoren $p_1(t)=t$ und $p_2(t)=1+t^2$ als Linearkombination von Elementen aus $\mathcal{A}$.     \end{itemize}

Avatar von

2 Antworten

0 Daumen
 
Beste Antwort

An dieser Stelle würde ich die Polynome einfach als Vektoren darstellen und diese auf lineare unabhängigkeit überprüfen.

Jup, du kannst bspw. bei a) \(V\) mit \(\mathbb{F}_2^3\) identifizieren und bei b) \(W\) mit \(\mathbb{R}^3\).

Nun noch zeigen, dass es sich um ein Erzeugendessytem handelt und der Teil der Aufgabe wäre doch erledigt, oder etwa nicht?

Falls klar ist, welche Dimension der UVR bezüglich des vorliegenden Körpers hat, dann kannst du es dir auch sparen.

Habe ich irgendetwas besonderes zu beachten, wenn ich zeigen will, dass es sich um eine Basis eines Untervektorraums handelt, anstatt um eine basis des vektorraums?

Nein, UVR sind ja auch Vektorräume.

Handelt es sich bei F2 um die Rechenvorschrift modulo 2? 

Nein, es handelt sich um einen Körper und nicht um eine Rechenvorschrift. Es ist der zweielementige Körper, der oft mit Hilfe der Restklassen modulo 2 repräsentiert wird.

Gruß

Avatar von 23 k

Erstmal vielen Dank :)

Dann lag ich mit meinen Überlegungen ja fast richtig.

Aufgabe b) habe ich soweit auch schon fertig. Nur bei a) habe ich noch ein paar Probleme mit dem Körper.

Es ist der zweielementige Körper, der oft mit Hilfe der Restklassen modulo 2 repräsentiert wird

Ich kann mir darunter irgendwie nichts vorstellen.

Wie habe ich mit der 1‾ zu rechnen?

Gruß

Der Körper besteht nur aus den Restklassen modulo 2 sprich beinhaltet nur die Elemente 0 und 1 (neutrale Elemente bzgl. Addition und Multiplikation). Schau bitte in deinen eigenen Unterlagen nach bzw. verifiziere für dich nochmal wie dieser Körper aussieht.

Der Beweis zu b) unterscheidet sich nicht, ist sogar einfacher, da die Anzahl möglicher Linearkombinationen endlich und übersichtlich ist.

Okay ich werde mal schauen :)

Trotzdem nochmal Danke :)

0 Daumen

Handelt es sich bei F2 um die Rechenvorschrift modulo 2?

Ja. Und jeder Unterraum ist ja selbst ein Vektorraum.

Deine Polynome sehen so aus a + bt + ct^2 mit   a,b,c aus F2 

und die kannst du mit

a*(1+t) + (b-a)*(t+t^2) + (c+a-b) *t^2   mit deinen "Basispolynomen"

erzeugen und die drei sind lin. unabhängig, denn

wenn  a*(1+t) + b*(t+t^2) + c *t^2  = 0-Polynom ist, dann gilt

a*1 + (a+b)*t + (b+c)*t^2  = 0-Polynom   und alle Koeffizienten = 0

bedeutet a= b=c =0

p(t) = 1*(1+t) + 1*(1+t^2) + 0*t^2 ist die Darstellung.

bei b so ähnlich und für die gesuchten Linearkombinationen

kannst du einen Ansatz der Art

t = a*(1+t+t^2)+b*(2t-t^2)+c*(2+t+t^2) machen und dann

  = (a+2c)*1 + ( a+2b+c)*t + (a-b+c)t^2

und zum Koeffizientenvergleich löst du

a+2c=0         a+2b+c=1       a+b+c=0

und hast die gesuchten Faktoren.

Avatar von 287 k 🚀

Ich danke dir für die ausführliche Antwort :)
Eine Frage habe ich aber noch. Was meinst du mit, bzw. wie kommst du darauf?

p(t) = 1*(1+t) + 1*(1+t2) + 0*t2 ist die Darstellung.

Ein anderes Problem?

Stell deine Frage

Willkommen bei der Mathelounge! Stell deine Frage einfach und kostenlos

x
Made by a lovely community